(3 intermediate revisions by 2 users not shown)
Line 1: Line 1:
= [[ECE PhD Qualifying Exams|ECE Ph.D. Qualifying Exam]] in "Communication, Networks, Signal, and Image Processing" (CS)  =
+
[[Category:ECE]]
 +
[[Category:QE]]
 +
[[Category:CNSIP]]
 +
[[Category:problem solving]]
 +
[[Category:image processing]]
  
= [[ECE-QE_CS5-2011|Question 1, August 2011]], Part 1 =
+
<center>
 +
<font size= 4>
 +
[[ECE_PhD_Qualifying_Exams|ECE Ph.D. Qualifying Exam]]
 +
</font size>
  
:[[ECE-QE_CS5-2011_solusion-1|Part 1]],[[ECE-QE CS1-2011 solusion-2|2]]]
+
<font size= 4>
 +
Communication, Networking, Signal and Image Processing (CS)
 +
 
 +
Question 5: Image Processing
 +
</font size>
 +
 
 +
August 2011
 +
</center>
 +
----
 +
----
 +
=Part 2=
 +
Jump to [[ECE-QE_CS5-2011_solusion-1|Part 1]],[[ECE-QE CS1-2011 solusion-2|2]]
  
 
----
 
----
Line 358: Line 376:
  
 
<font face="serif"><span style="font-size: 19px;"><math>\color{red}
 
<font face="serif"><span style="font-size: 19px;"><math>\color{red}
5. \text{ Back project } g_{\theta}(r) \text{ and get: }
+
5. \text{ Back project } g_{\theta}(r) \text{ (obtained from step 4) and get: }
 
</math></span></font>  
 
</math></span></font>  
 
   
 
   
Line 384: Line 402:
 
f(x,y) = \int_{0}^{\pi}{g_\theta(xcos\theta + ysin\theta)d\theta}
 
f(x,y) = \int_{0}^{\pi}{g_\theta(xcos\theta + ysin\theta)d\theta}
 
</math></span></font>  
 
</math></span></font>  
 +
 +
----
 +
=Solution 3:=
 +
 +
a)
 +
 +
<math>p_\theta(r)=\begin{cases}1, & \text{if } r=0\\0, & \text{otherwisr}\end{cases}=\delta(r)</math>.
 +
 +
[[Image:CW_QE_2011_CS5_2_a_sol3.png|200px]]
 +
 +
 +
b)
 +
 +
* If <math>0\leq\theta<\frac{\pi}{4}</math>, <math>d=\sqrt{2}\text{cos}(\frac{\pi}{4}-\theta)=\sqrt{2}[\text{cos}\frac{\pi}{4}\text{cos}\theta+\text{sin}\frac{\pi}{4}\text{sin}\theta]=\text{cos}\theta+\text{sin}\theta</math>.
 +
 +
&nbsp;&nbsp;&nbsp;&nbsp;&nbsp;&nbsp;&nbsp;&nbsp;[[Image:CW_QE_2011_CS5_2_b_1_sol3.png|300px]]
 +
 +
* If <math>\frac{\pi}{4}\leq \theta < \frac{\pi}{2}</math>, <math>d=\sqrt{2}\text{cos}(\theta-\frac{\pi}{4})=\sqrt{2}\text{cos}(\frac{\pi}{4}-\theta)=\text{cos}\theta+\text{sin}\theta</math>.
 +
 +
&nbsp;&nbsp;&nbsp;&nbsp;&nbsp;&nbsp;&nbsp;&nbsp;[[Image:CW_QE_2011_CS5_2_b_2_sol3.png|300px]]
 +
 +
* If <math>\frac{\pi}{2}\leq \theta < \frac{3\pi}{4}</math>, <math>d=\sqrt{2}\text{sin}(\frac{\pi}{4}-(\theta-\frac{\pi}{2}))=\sqrt{2}\text{sin}(\frac{3\pi}{4}-\theta)=\sqrt{2}[\text{sin}\frac{3\pi}{4}\text{cos}\theta-\text{cos}\frac{3\pi}{4}\text{sin}\theta]=\text{cos}\theta+\text{sin}\theta</math>.
 +
 +
&nbsp;&nbsp;&nbsp;&nbsp;&nbsp;&nbsp;&nbsp;&nbsp;[[Image:CW_QE_2011_CS5_2_b_3_sol3.png|300px]]
 +
 +
* If <math>\frac{3\pi}{4}\leq \theta < 2\pi</math>, <math>d=-\sqrt{2}\text{sin}(\theta-\frac{\pi}{2}-\frac{\pi}{4})=-\sqrt{2}\text{sin}(\theta-\frac{3\pi}{4})=-\sqrt{2}[\text{sin}\theta\text{cos}\frac{3\pi}{4}-\text{cos}\theta\text{sin}\frac{3\pi}{4}]=\text{cos}\theta+\text{sin}\theta</math>.
 +
 +
&nbsp;&nbsp;&nbsp;&nbsp;&nbsp;&nbsp;&nbsp;&nbsp;[[Image:CW_QE_2011_CS5_2_b_4_sol3.png|300px]]
 +
 +
<math>\Rightarrow p_\theta(r)=\delta(r-(\text{cos}\theta+\text{sin}\theta))</math> for all <math>\theta</math>.
 +
 +
 +
c)
 +
 +
Since <math>f(x,y)</math> is symmetric to <math>(0,0)</math>
 +
 +
<math>\Rightarrow p_\theta(r)=p_0(r)=\int_{-\sqrt{\frac{1}{4}-r^2}}^{\sqrt{\frac{1}{4}-r^2}}1\cdot dz=2\int_0^{\sqrt{\frac{1}{4}-r^2}}1\cdot dz\text{, for }\vert r\vert<\frac{1}{2}</math>
 +
 +
[[Image:CW_QE_2011_CS5_2_c_sol3.png|200px]]
 +
 +
<math>\Rightarrow p_\theta(r)=2\sqrt{\frac{1}{4}-r^2}\text{rect}(r)</math>
 +
 +
 +
d)
 +
 +
From part (b), we know that, if <math>f(x,y)</math> has <math>(1,1)</math> shift in <math>x-y</math> plane, the <math>p_\theta(r)</math> would have <math>(\text{cos}\theta+\text{sin}\theta)</math> shift in <math>r</math>-direction.
 +
 +
<math>\Rightarrow p_\theta(r)=2\sqrt{\frac{1}{4}-(r-(\text{cos}\theta+\text{sin}\theta))^2}\text{rect}(r-(\text{cos}\theta+\text{sin}\theta))</math>
 +
 +
 +
e)
 +
 +
* First, measure the forward projection <math>p_\theta(r)</math> for sufficient angles <math>\theta s</math>.
 +
* Second, filter the forward projection with <math>h(r): g_\theta(r)=h(r)\ast p_\theta(r)</math>, where <math>h(r)=\text{CTFT}^{-1}\left\{\vert\rho\vert\right\}</math>.
 +
* Finally, back project filtered projection to reconstruct <math>f(x,y)</math>. <math>f(x,y)=\int_0^\pi g_\theta(x\text{cos}\theta+y\text{sin}\theta)d\theta</math>.
 +
 +
 +
 +
=Solution 4:=
 +
 +
a)
 +
 +
<math>P_\theta(\rho)=F(\rho\text{cos}\theta, \rho\text{sin}\theta)</math>
 +
 +
When <math>f(x,y)=\delta(x,y)</math>, <math>F(\mu,\nu)=1</math>.
 +
 +
So <math>p_\theta(r)=\delta_\theta(r)</math>.
 +
 +
<font color="green"><u>'''Comments'''</u>:
 +
 +
* The answer would be easier to understand if the last statement was expressed in this way: <math>p_{\theta}(r)=\mathcal{F}^{-1}\{ P_{\theta}(\rho)\}=\mathcal{F}^{-1}\{1\}=\delta_{\theta}(r)</math>, where <math>\mathcal{F}^{-1}\{\cdot\}</math> is the Fourier inverse transform.
 +
 +
</font>
 +
 +
 +
b)
 +
 +
When <math>f(x,y)=\delta(x-1,y-1)</math>, <math>F(\mu,\nu)=e^{-j\mu}\cdot e^{-j\nu}</math>.
 +
 +
So <math>P_\theta(\rho)=e^{-j\rho\text{cos}\theta}\cdot e^{-j\rho\text{sin}\theta}</math>, <math>p_\theta(r)=\delta(r-\text{cos}\theta-\text{sin}\theta)</math>.
 +
 +
 +
c)
 +
 +
[[Image:CW_QE_2011_CS5_2_c_sol4.png|200px]]
 +
 +
<math>p_\theta(r)=\int_{-\sqrt{\frac{1}{4}-r^2}}^{\sqrt{\frac{1}{4}-r^2}}1dz=\sqrt{1-4r^2}</math>
 +
 +
So <math>p_\theta(r)=\begin{cases} \sqrt{1-4r^2}, & |r|<\frac{1}{2}\\0, & |r|>\frac{1}{2}\end{cases}</math>.
 +
 +
 +
d)
 +
 +
[[Image:CW_QE_2011_CS5_2_d_sol4.png|300px]]
 +
 +
<math>r_0=\sqrt{2}\cdot\left(\frac{\pi}{4}-\theta\right)</math>
 +
 +
<math>p_\theta(r)=\int_{-\infty}^{\infty}f(r\text{cos}\theta-z\text{sin}\theta, r\text{sin}\theta+z\text{cos}\theta)dz=\int_{-\sqrt{\frac{1}{4}-(r-r_0)^2}}^{\sqrt{\frac{1}{4}-(r-r_0)^2}}adz</math>
 +
 +
So <math>p_\theta(r)=\begin{cases}\sqrt{1-4\left(r-\sqrt{2}\left(\frac{\pi}{4}-\theta\right)\right)}, & |r-r_0|\leq\frac{1}{2}\\0, & \text{otherwise}\end{cases}</math>.
 +
 +
<font color="green"><u>'''Comments'''</u>:
 +
 +
* The computation for <math>r_0</math> is wrong. The correct expression of <math>r_0</math> should be: <math>r_0 = \sqrt{2}\sin(\frac{\pi}{4}-\theta)</math>. It can also be expressed as: <math>r_0=\cos\theta+\sin\theta</math>.
 +
* The rest statement is correct, and then the final result becomes: <math>p_{\theta}(r)=\begin{cases}\sqrt{1-4(r-r_0)}, & |r-r_0|\leq\frac{1}{2}\\0, & \text{otherwise} \end{cases}</math>.
 +
 +
</font>
 +
 +
 +
e)
 +
 +
<math>
 +
\begin{align}
 +
f(x,y) & =\int_{-\infty}^{\infty}\int_{-\infty}^{\infty}F(\mu,\nu)e^{2\pi j(x\mu+y\nu)}d\mu d\nu\\
 +
& = \int_{-\infty}^{\infty}\int_{0}^{\pi}p_\theta(\rho)e^{2\pi j(x\rho\text{cos}\theta+y\rho\text{sin}\theta)}|\rho|d\theta d\rho\\
 +
& = \int_{0}^{\pi}\underbrace{\int_{-\infty}^{\infty}|\rho|p_\theta(\rho)e^{2\pi j(x\rho\text{cos}\theta+y\rho\text{sin}\theta)}d\rho}_{g_\theta(x\text{cos}\theta+y\text{sin}\theta)}d\theta
 +
\end{align}
 +
</math>
 +
 +
So <math>g_\theta(t)</math> is given by:
 +
 +
<math>g_\theta(t)=\int_{-\infty}^{\infty}|\rho|p_\theta(\rho)e^{2\pi j\rho t}d\rho=\text{CTFT}^{-1}\{|\rho|p_\theta(\rho)\}=h(t)\ast p_\theta(r)</math>
 +
 +
So what we do is:
 +
 +
# measure projection <math>p_\theta(r)</math>;
 +
# filter the projection <math>g_\theta(r)=h(r)\ast p_\theta(r)</math>;
 +
# back project filtered projection: <math>f(x,y)=\int_0^{\pi}g_\theta(x\text{cos}\theta)+y\text{sin}\theta)d\theta</math>.
 +
 +
 +
----
 +
=Related Problems=
 +
 +
a) Calculate the forward projection <math>p_\theta(r)</math>, for <math>f(x,y)=\text{rect}(x,y)</math>, as <math>\theta=0</math> and <math>\theta=\pi/4</math>.
 +
 +
b) Calculate the forward projection <math>p_\theta(r)</math>, for <math>f(x,y)=\delta(x-a,y-b)</math>.
  
 
----
 
----
Line 396: Line 550:
 
----
 
----
  
[[ECE PhD Qualifying Exams|Back to ECE Qualifying Exams (QE) page]]
+
[[ECE PhD Qualifying Exams|Back to ECE Qualifying Exams (QE) page]]
 
+
[[Category:ECE]] [[Category:QE]] [[Category:Automatic_Control]] [[Category:Problem_solving]]
+

Latest revision as of 01:51, 31 March 2015


ECE Ph.D. Qualifying Exam

Communication, Networking, Signal and Image Processing (CS)

Question 5: Image Processing

August 2011



Part 2

Jump to Part 1,2


 $ \color{blue}\text{Consider an image } f(x,y) \text{ with a forward projection} $

                $ \color{blue} p_{\theta}(r) = \mathcal{FP}\left \{ f(x,y) \right \} $

                             $ \color{blue} = \int_{-\infty}^{\infty}{f \left ( r cos(\theta) - z sin(\theta),r sin(\theta) + z cos(\theta) \right )dz}. $

$ \color{blue} \text{Let } F(\mu,\nu) \text{ be the continuous-time Fourier transform of } f(x,y) \text{ given by} $
              $ \color{blue} F(u,v) = \int_{-\infty}^{\infty}{\int_{-\infty}^{\infty}{f(x,y)e^{-j2\pi(ux,vy)}dx}dy} $

$ \color{blue} \text{and let } P_{\theta}(\rho) \text{ be the continuous-time Fourier transform of } p_{\theta}(r) \text{ given by} $
              $ \color{blue} P_{\theta}(\rho) = \int_{-\infty}^{\infty}{p_{\theta}(r)e^{-j2\pi(\rho r)}dr}. $


$ \color{blue}\text{a) Calculate the forward projection }p_{\theta}(r) \text{, for } f(x,y) = \delta(x,y). $

$ \color{blue}\text{Solution 1:} $

$ p_{\theta}(r) = \int_{-\infty}^{+\infty}{\delta(r cos\theta - z sin\theta, r sin\theta + z cos \theta) dz} $


$ = z \text{ when } \left\{\begin{matrix} r cos \theta - z sin \theta = 0 \\ r sin \theta + z cos \theta = 0 \end{matrix}\right. $


$ = \frac{r cos\theta}{sin \theta}, \theta > 0 $

$ {\color{red} \text{This answer is incorrect. The correct answer is as following:}} $

$ {\color{green} \text{Recall:}} $


$ {\color{green} \text{i) } \int_{-\infty}^{+\infty}{f(g(t)) \delta (t) dt} = f(g(t=0)) \int_{-\infty}^{+\infty}{\delta (t) dt} } $


$ {\color{green} \text{ii) } \int_{-\infty}^{+\infty}{\delta (\alpha t) dt} = \int_{-\infty}^{+\infty}{\delta (u) \frac{du}{|\alpha|}} = \frac{1}{|\alpha|} } $


$ {\color{green} \text{iii) } \delta() \text{ function is separable: } \delta(x,y) = \delta(x) \cdot \delta(y) } $


$ \color{green} \text{ Define } u = r cos\theta - z sin\theta $


$ \color{green} \Rightarrow dz = \frac{du}{|sin\theta|} $


$ \color{green} \text{ Now } $


$ \color{green} p_{\theta}(r) = \int_{-\infty}^{+\infty}{\delta(r cos\theta - z sin\theta, r sin\theta + z cos \theta) dz} $


$ \color{green} p_{\theta}(r) = \int_{-\infty}^{+\infty}{\delta(g(u)) \delta(u) \frac{du}{|sin\theta|}} = \frac{\delta(u=0)}{|sin\theta|} $


$ \color{green} = \frac{\delta(\frac{r}{sin\theta})}{|sin\theta|} = \frac{|sin\theta|}{|sin\theta|} \delta(r) = \delta(r) $


$ \color{blue}\text{Solution 2:} $

.QE 11 CS5 2 a sol2.PNG

$ p_{\theta}(r) = \int_{-\infty}^{+\infty}{\delta(r cos\theta - z sin\theta, r sin\theta + z cos \theta) dz} $


$ = \delta(r) $


$ {\color{green} \text{Here, the student uses the intuitive solution: in this case the answer does not depend on } \theta \text{, since the image just contains a peak at origin. } } $


$ \color{blue}\text{b) Calculate the forward projection }p_{\theta}(r) \text{, for } f(x,y) = \delta(x-1,y-1). $

$ \color{blue}\text{Solution 1:} $

$ p_{\theta}(r) = \int_{-\infty}^{+\infty}{\delta(r cos\theta - z sin\theta - 1, r sin\theta + z cos \theta - 1) dz} $

$ = z \text{ when } \left\{\begin{matrix} r cos \theta - z sin \theta = 1 \\ r sin \theta + z cos \theta = 1 \end{matrix}\right. $


$ = \frac{r cos\theta - 1}{sin \theta}, \theta > 0 $

$ {\color{red} \text{This answer is incorrect. The correct answer is as following:}} $

$ \color{green} \text{ Similar to the solution 1 to part a) we define u: } u = r cos\theta - z sin\theta - 1 $


$ \color{green} \text{ Following the same logic as in part a) we obtain the final answer:} $


$ \color{green} p_{\theta}(r) = \delta(r - (cos\theta + sin \theta)) = \delta(r - \sqrt{2} cos (\theta - \frac{\pi}{4})) $


$ \color{blue}\text{Solution 2:} $

QE 11 CS5 2 b sol2.PNG

$ \tilde{p}_\theta(r) = p_{\theta}(r - \sqrt{1+1} cos(\theta - tan^{-1}(\frac{1}{1}))) $


$ = p_\theta(r - \sqrt{2} cos(\theta - \frac{\pi}{4})) $


$ = \delta(r - \sqrt{2} cos(\theta - \frac{\pi}{4})) $


$ {\color{green} \text{Again, the student uses the intuitive solution: in this case the answer does depend on } \theta \text{, since the peak is shifted from the origin to the point } (1,1). } $



$ \color{blue}\text{c) Calculate the forward projection }p_{\theta}(r) \text{, for } f(x,y) = rect \left(\sqrt[]{x^2+y^2} \right). $


$ \color{blue}\text{Solution 1:} $


$ p_{\theta}(r) = \int_{-\infty}^{+\infty}{rect(\sqrt{(r cos\theta - z sin\theta)^2 + (r sin\theta + z cos \theta)^2)} dz} $


$ \color{green} \text{Recall should be added:} $

$ \color{green} rect(t) = \left\{\begin{matrix} 1, for |t|\leq \frac{1}{2} \\ 0, otherwise \end{matrix}\right. $


$ \color{green} \text{therefore: } $


$ p_{\theta}(r) = \int_{-\sqrt{\frac{1}{4} - r^2}}^{\sqrt{\frac{1}{4} - r^2}}{1 dz} $


$ = \left\{\begin{matrix} \sqrt{1 - 4r^2}, &\text{ if }|r| \leq \frac{1}{2} \\ 0, &\text{ otherwise} \end{matrix}\right. $


$ \color{blue}\text{Solution 2:} $

QE 11 CS5 2 c sol2.PNG

$ p_{\theta}(r) = \int_{-\infty}^{+\infty}{f(r cos\theta - z sin\theta, r sin\theta + z cos \theta) dz} $


$ = \int_{-\sqrt{\frac{1}{4} - r^2}}^{\sqrt{\frac{1}{4} - r^2}}{1 dz} = \sqrt{1 - 4r^2}, \text{ if }|r| \leq \frac{1}{2} $


$ \text{ else } p_{\theta}(r) = 0 $


$ \color{blue}\text{d) Calculate the forward projection }p_{\theta}(r) \text{, for } f(x,y) = rect \left(\sqrt[]{(x-1)^2+(y-1)^2} \right). $

$ \color{blue}\text{Solution 1:} $


$ p_{\theta}(r) = \int_{-\infty}^{+\infty}{rect \left( \sqrt{(r cos\theta - z sin\theta - 1)^2 + (r sin\theta + z cos \theta - 1)^2} \right) dz} $


$ = \int_{-\sqrt{\frac{1}{4} - (r - (cos\theta + sin\theta))^2}}^{\sqrt{\frac{1}{4} - (r - (cos\theta + sin\theta))^2}}{1 dz} $


$ = \left\{\begin{matrix} \sqrt{1 - 4(r - (cos\theta + sin\theta))^2}, &\text{ if }|r| \leq \frac{1}{2} \\ 0, &\text{ otherwise} \end{matrix}\right. $


$ \color{green} \text{ To make it more clear, the following form could be obtained:} $

$ \color{green} = \left\{\begin{matrix} \sqrt{1 - 4(r - \sqrt{2} cos (\theta - \frac{\pi}{4}))^2}, &\text{ if }|r| \leq \frac{1}{2} \\ 0, &\text{ otherwise} \end{matrix}\right. $


$ \color{blue}\text{Solution 2:} $

$ \tilde{p}_\theta(r) = p_{\theta}(r - \sqrt{1+1} cos(\theta - tan^{-1}(\frac{1}{1}))) $


$ = p_\theta(r - \sqrt{2} cos(\theta - \frac{\pi}{4})) $


$ \text{ where } p_\theta(r) = \left\{\begin{matrix} \sqrt{1 - 4r^2}, &\text{ if }|r| \leq \frac{1}{2} \\ 0, &\text{ else} \end{matrix}\right. $


$ {\color{green} \text{Here, the student uses the results from solutions to part b and c.} } $


$ \color{blue}\text{e) Describe in precise detail, the steps required to perform filtered back projection (FBP) reconstruction of } f(x,y). $


$ \color{blue}\text{Solution 1:} $

$ 1. \text{ Compute } \rho_{\theta}(r) $


$ 2. \text{ Compute FT of step 1.} $

$ 3. \text{ Multiply step 2 by the filter } H(\rho) = |\rho| = f_c \left [ rect(\frac{f}{2f_c}) - \Lambda(\frac{f}{f_c}) \right ] $

$ 4. \text{ Compute inverseFT of step 3.} $

$ \color{red} \text{ This answer is not complete. We need the final step:} $


$ \color{red} 5. \text{ Back project } g_{\theta}(r) \text{ (obtained from step 4) and get: } $

$ \color{red} f(x,y) = \int_{0}^{\pi}{g_\theta(xcos\theta + ysin\theta)d\theta} $


$ \color{blue}\text{Solution 2:} $

$ 1. \text{ Measure the projections } \rho_{\theta}(r) \text{ at various angles} $

$ 2. \text{ Filter the projections } \rho_{\theta}(r) \text{ with } h(r) \text{, where } H(\rho) = |\rho| \text{ and get } g_{\theta}(r) $

$ 3. \text{ Back project } g_{\theta}(r) \text{ along } r = xcos\theta + ysin\theta \text{ and get } $

$ f(x,y) = \int_{0}^{\pi}{g_\theta(xcos\theta + ysin\theta)d\theta} $


Solution 3:

a)

$ p_\theta(r)=\begin{cases}1, & \text{if } r=0\\0, & \text{otherwisr}\end{cases}=\delta(r) $.

CW QE 2011 CS5 2 a sol3.png


b)

  • If $ 0\leq\theta<\frac{\pi}{4} $, $ d=\sqrt{2}\text{cos}(\frac{\pi}{4}-\theta)=\sqrt{2}[\text{cos}\frac{\pi}{4}\text{cos}\theta+\text{sin}\frac{\pi}{4}\text{sin}\theta]=\text{cos}\theta+\text{sin}\theta $.

        CW QE 2011 CS5 2 b 1 sol3.png

  • If $ \frac{\pi}{4}\leq \theta < \frac{\pi}{2} $, $ d=\sqrt{2}\text{cos}(\theta-\frac{\pi}{4})=\sqrt{2}\text{cos}(\frac{\pi}{4}-\theta)=\text{cos}\theta+\text{sin}\theta $.

        CW QE 2011 CS5 2 b 2 sol3.png

  • If $ \frac{\pi}{2}\leq \theta < \frac{3\pi}{4} $, $ d=\sqrt{2}\text{sin}(\frac{\pi}{4}-(\theta-\frac{\pi}{2}))=\sqrt{2}\text{sin}(\frac{3\pi}{4}-\theta)=\sqrt{2}[\text{sin}\frac{3\pi}{4}\text{cos}\theta-\text{cos}\frac{3\pi}{4}\text{sin}\theta]=\text{cos}\theta+\text{sin}\theta $.

        CW QE 2011 CS5 2 b 3 sol3.png

  • If $ \frac{3\pi}{4}\leq \theta < 2\pi $, $ d=-\sqrt{2}\text{sin}(\theta-\frac{\pi}{2}-\frac{\pi}{4})=-\sqrt{2}\text{sin}(\theta-\frac{3\pi}{4})=-\sqrt{2}[\text{sin}\theta\text{cos}\frac{3\pi}{4}-\text{cos}\theta\text{sin}\frac{3\pi}{4}]=\text{cos}\theta+\text{sin}\theta $.

        CW QE 2011 CS5 2 b 4 sol3.png

$ \Rightarrow p_\theta(r)=\delta(r-(\text{cos}\theta+\text{sin}\theta)) $ for all $ \theta $.


c)

Since $ f(x,y) $ is symmetric to $ (0,0) $

$ \Rightarrow p_\theta(r)=p_0(r)=\int_{-\sqrt{\frac{1}{4}-r^2}}^{\sqrt{\frac{1}{4}-r^2}}1\cdot dz=2\int_0^{\sqrt{\frac{1}{4}-r^2}}1\cdot dz\text{, for }\vert r\vert<\frac{1}{2} $

CW QE 2011 CS5 2 c sol3.png

$ \Rightarrow p_\theta(r)=2\sqrt{\frac{1}{4}-r^2}\text{rect}(r) $


d)

From part (b), we know that, if $ f(x,y) $ has $ (1,1) $ shift in $ x-y $ plane, the $ p_\theta(r) $ would have $ (\text{cos}\theta+\text{sin}\theta) $ shift in $ r $-direction.

$ \Rightarrow p_\theta(r)=2\sqrt{\frac{1}{4}-(r-(\text{cos}\theta+\text{sin}\theta))^2}\text{rect}(r-(\text{cos}\theta+\text{sin}\theta)) $


e)

  • First, measure the forward projection $ p_\theta(r) $ for sufficient angles $ \theta s $.
  • Second, filter the forward projection with $ h(r): g_\theta(r)=h(r)\ast p_\theta(r) $, where $ h(r)=\text{CTFT}^{-1}\left\{\vert\rho\vert\right\} $.
  • Finally, back project filtered projection to reconstruct $ f(x,y) $. $ f(x,y)=\int_0^\pi g_\theta(x\text{cos}\theta+y\text{sin}\theta)d\theta $.


Solution 4:

a)

$ P_\theta(\rho)=F(\rho\text{cos}\theta, \rho\text{sin}\theta) $

When $ f(x,y)=\delta(x,y) $, $ F(\mu,\nu)=1 $.

So $ p_\theta(r)=\delta_\theta(r) $.

Comments:

  • The answer would be easier to understand if the last statement was expressed in this way: $ p_{\theta}(r)=\mathcal{F}^{-1}\{ P_{\theta}(\rho)\}=\mathcal{F}^{-1}\{1\}=\delta_{\theta}(r) $, where $ \mathcal{F}^{-1}\{\cdot\} $ is the Fourier inverse transform.


b)

When $ f(x,y)=\delta(x-1,y-1) $, $ F(\mu,\nu)=e^{-j\mu}\cdot e^{-j\nu} $.

So $ P_\theta(\rho)=e^{-j\rho\text{cos}\theta}\cdot e^{-j\rho\text{sin}\theta} $, $ p_\theta(r)=\delta(r-\text{cos}\theta-\text{sin}\theta) $.


c)

CW QE 2011 CS5 2 c sol4.png

$ p_\theta(r)=\int_{-\sqrt{\frac{1}{4}-r^2}}^{\sqrt{\frac{1}{4}-r^2}}1dz=\sqrt{1-4r^2} $

So $ p_\theta(r)=\begin{cases} \sqrt{1-4r^2}, & |r|<\frac{1}{2}\\0, & |r|>\frac{1}{2}\end{cases} $.


d)

CW QE 2011 CS5 2 d sol4.png

$ r_0=\sqrt{2}\cdot\left(\frac{\pi}{4}-\theta\right) $

$ p_\theta(r)=\int_{-\infty}^{\infty}f(r\text{cos}\theta-z\text{sin}\theta, r\text{sin}\theta+z\text{cos}\theta)dz=\int_{-\sqrt{\frac{1}{4}-(r-r_0)^2}}^{\sqrt{\frac{1}{4}-(r-r_0)^2}}adz $

So $ p_\theta(r)=\begin{cases}\sqrt{1-4\left(r-\sqrt{2}\left(\frac{\pi}{4}-\theta\right)\right)}, & |r-r_0|\leq\frac{1}{2}\\0, & \text{otherwise}\end{cases} $.

Comments:

  • The computation for $ r_0 $ is wrong. The correct expression of $ r_0 $ should be: $ r_0 = \sqrt{2}\sin(\frac{\pi}{4}-\theta) $. It can also be expressed as: $ r_0=\cos\theta+\sin\theta $.
  • The rest statement is correct, and then the final result becomes: $ p_{\theta}(r)=\begin{cases}\sqrt{1-4(r-r_0)}, & |r-r_0|\leq\frac{1}{2}\\0, & \text{otherwise} \end{cases} $.


e)

$ \begin{align} f(x,y) & =\int_{-\infty}^{\infty}\int_{-\infty}^{\infty}F(\mu,\nu)e^{2\pi j(x\mu+y\nu)}d\mu d\nu\\ & = \int_{-\infty}^{\infty}\int_{0}^{\pi}p_\theta(\rho)e^{2\pi j(x\rho\text{cos}\theta+y\rho\text{sin}\theta)}|\rho|d\theta d\rho\\ & = \int_{0}^{\pi}\underbrace{\int_{-\infty}^{\infty}|\rho|p_\theta(\rho)e^{2\pi j(x\rho\text{cos}\theta+y\rho\text{sin}\theta)}d\rho}_{g_\theta(x\text{cos}\theta+y\text{sin}\theta)}d\theta \end{align} $

So $ g_\theta(t) $ is given by:

$ g_\theta(t)=\int_{-\infty}^{\infty}|\rho|p_\theta(\rho)e^{2\pi j\rho t}d\rho=\text{CTFT}^{-1}\{|\rho|p_\theta(\rho)\}=h(t)\ast p_\theta(r) $

So what we do is:

  1. measure projection $ p_\theta(r) $;
  2. filter the projection $ g_\theta(r)=h(r)\ast p_\theta(r) $;
  3. back project filtered projection: $ f(x,y)=\int_0^{\pi}g_\theta(x\text{cos}\theta)+y\text{sin}\theta)d\theta $.



Related Problems

a) Calculate the forward projection $ p_\theta(r) $, for $ f(x,y)=\text{rect}(x,y) $, as $ \theta=0 $ and $ \theta=\pi/4 $.

b) Calculate the forward projection $ p_\theta(r) $, for $ f(x,y)=\delta(x-a,y-b) $.


"Communication, Networks, Signal, and Image Processing" (CS)- Question 5, August 2011

Go to


Back to ECE Qualifying Exams (QE) page

Alumni Liaison

Abstract algebra continues the conceptual developments of linear algebra, on an even grander scale.

Dr. Paul Garrett